Why $int_2^infty{1over{sqrt x(sqrt x-1)^2}}dx style{text-decoration:line-through}{approx}...

How does one intimidate enemies without having the capacity for violence?

Minkowski space

Dragon forelimb placement

Has the BBC provided arguments for saying Brexit being cancelled is unlikely?

Why dont electromagnetic waves interact with each other?

Problem of parity - Can we draw a closed path made up of 20 line segments...

Can a Warlock become Neutral Good?

Why does Kotter return in Welcome Back Kotter?

How is it possible to have an ability score that is less than 3?

What does it mean to describe someone as a butt steak?

What does "Puller Prush Person" mean?

A newer friend of my brother's gave him a load of baseball cards that are supposedly extremely valuable. Is this a scam?

Theorems that impeded progress

How can bays and straits be determined in a procedurally generated map?

Email Account under attack (really) - anything I can do?

Can an x86 CPU running in real mode be considered to be basically an 8086 CPU?

TGV timetables / schedules?

Why "Having chlorophyll without photosynthesis is actually very dangerous" and "like living with a bomb"?

Approximately how much travel time was saved by the opening of the Suez Canal in 1869?

Is it possible to do 50 km distance without any previous training?

Do I have a twin with permutated remainders?

The use of multiple foreign keys on same column in SQL Server

Why Is Death Allowed In the Matrix?

Font hinting is lost in Chrome-like browsers (for some languages )



Why $int_2^infty{1over{sqrt x(sqrt x-1)^2}}dx style{text-decoration:line-through}{approx} sum_{n=2}^{infty}{1over{sqrt n(sqrt n-1)^2}}$?


Why does the Euler-Maclaurin summation formula not exist in $2$ or more dimensions?Estimating the sum $sum_{k=2}^{infty} frac{1}{k ln^2(k)}$How close can $sum_{k=1}^n sqrt{k}$ be to an integer?Proof that $sumlimits_{i=1}^n cos sqrt{i}$ is unbounded.Turning infinite sum into integralWhat is $lim_{xto infty} 2sqrt{x}- sum_{n=1}^x {1over sqrt{n}}$?Prove that: $zeta(3)=lim_{Nto infty}{1over N}sum_{k=1}^{N}{1over k^{phi^2}ln{left(1+{k^{phi^{-2}}over N}right)}}$What is $sum_{n=1}^{infty} frac{1}{sqrt{n^{3} + 1}}$?Is there a closed form for $sum_{n=0}^{infty}{2^{n+1}over {2n choose n}}cdotleft({2n-1over 2n+1}right)^2?$Euler-Maclaurin Formula Definition Confusion













0












$begingroup$


I am aware that there are certain cases where the infinite sum does not equal the infinite integral. However, I am not yet advanced enough to be able to understand the Euler–Maclaurin formula, especially in terms of coming up with the $k$th Bernoulli Number. That said, I am wondering if there is a way somebody could explain to me the following inequality that came up in my course. When I use a calculator to estimate the infinite sum $sum_{n=2}^{infty}{1over{sqrt n(sqrt n-1)^2}}$, I get $$sum_{n=2}^{9999999}{1over{sqrt n(sqrt n-1)^2}} approx 7.47356$$



When I try to estimate using an integral, I get a huge discrepancy
$$int_2^infty{1over{sqrt x(sqrt x-1)^2}}dx=2^{3over2}+2 approx 4.8284$$



Does this require the Euler–Maclaurin formula to explain, or is there an easier way to understand what is going on over here?










share|cite|improve this question









$endgroup$








  • 2




    $begingroup$
    You may want to take a look at this: en.wikipedia.org/wiki/Abel%E2%80%93Plana_formula
    $endgroup$
    – Zachary
    Mar 19 at 23:56
















0












$begingroup$


I am aware that there are certain cases where the infinite sum does not equal the infinite integral. However, I am not yet advanced enough to be able to understand the Euler–Maclaurin formula, especially in terms of coming up with the $k$th Bernoulli Number. That said, I am wondering if there is a way somebody could explain to me the following inequality that came up in my course. When I use a calculator to estimate the infinite sum $sum_{n=2}^{infty}{1over{sqrt n(sqrt n-1)^2}}$, I get $$sum_{n=2}^{9999999}{1over{sqrt n(sqrt n-1)^2}} approx 7.47356$$



When I try to estimate using an integral, I get a huge discrepancy
$$int_2^infty{1over{sqrt x(sqrt x-1)^2}}dx=2^{3over2}+2 approx 4.8284$$



Does this require the Euler–Maclaurin formula to explain, or is there an easier way to understand what is going on over here?










share|cite|improve this question









$endgroup$








  • 2




    $begingroup$
    You may want to take a look at this: en.wikipedia.org/wiki/Abel%E2%80%93Plana_formula
    $endgroup$
    – Zachary
    Mar 19 at 23:56














0












0








0





$begingroup$


I am aware that there are certain cases where the infinite sum does not equal the infinite integral. However, I am not yet advanced enough to be able to understand the Euler–Maclaurin formula, especially in terms of coming up with the $k$th Bernoulli Number. That said, I am wondering if there is a way somebody could explain to me the following inequality that came up in my course. When I use a calculator to estimate the infinite sum $sum_{n=2}^{infty}{1over{sqrt n(sqrt n-1)^2}}$, I get $$sum_{n=2}^{9999999}{1over{sqrt n(sqrt n-1)^2}} approx 7.47356$$



When I try to estimate using an integral, I get a huge discrepancy
$$int_2^infty{1over{sqrt x(sqrt x-1)^2}}dx=2^{3over2}+2 approx 4.8284$$



Does this require the Euler–Maclaurin formula to explain, or is there an easier way to understand what is going on over here?










share|cite|improve this question









$endgroup$




I am aware that there are certain cases where the infinite sum does not equal the infinite integral. However, I am not yet advanced enough to be able to understand the Euler–Maclaurin formula, especially in terms of coming up with the $k$th Bernoulli Number. That said, I am wondering if there is a way somebody could explain to me the following inequality that came up in my course. When I use a calculator to estimate the infinite sum $sum_{n=2}^{infty}{1over{sqrt n(sqrt n-1)^2}}$, I get $$sum_{n=2}^{9999999}{1over{sqrt n(sqrt n-1)^2}} approx 7.47356$$



When I try to estimate using an integral, I get a huge discrepancy
$$int_2^infty{1over{sqrt x(sqrt x-1)^2}}dx=2^{3over2}+2 approx 4.8284$$



Does this require the Euler–Maclaurin formula to explain, or is there an easier way to understand what is going on over here?







calculus integration sequences-and-series






share|cite|improve this question













share|cite|improve this question











share|cite|improve this question




share|cite|improve this question










asked Mar 19 at 23:40









agbltagblt

350114




350114








  • 2




    $begingroup$
    You may want to take a look at this: en.wikipedia.org/wiki/Abel%E2%80%93Plana_formula
    $endgroup$
    – Zachary
    Mar 19 at 23:56














  • 2




    $begingroup$
    You may want to take a look at this: en.wikipedia.org/wiki/Abel%E2%80%93Plana_formula
    $endgroup$
    – Zachary
    Mar 19 at 23:56








2




2




$begingroup$
You may want to take a look at this: en.wikipedia.org/wiki/Abel%E2%80%93Plana_formula
$endgroup$
– Zachary
Mar 19 at 23:56




$begingroup$
You may want to take a look at this: en.wikipedia.org/wiki/Abel%E2%80%93Plana_formula
$endgroup$
– Zachary
Mar 19 at 23:56










1 Answer
1






active

oldest

votes


















2












$begingroup$

The discrepancy comes mostly from the first two terms: the integral from $2$ to $3$ is about $2.1$ while the sum term for $n=2$ is about $4.2$ Same from $3$ to $4$ integral is about $.73$, the $n=3$ sum term is about $1.1$ so if you start from $4$ on and noting that because the function is decreasing, the integral is always smaller than the sum starting from same bound, you get the integral to be about $2$ and the sum to be about $2.17$ so the discrepancy gets much smaller.






share|cite|improve this answer









$endgroup$














    Your Answer





    StackExchange.ifUsing("editor", function () {
    return StackExchange.using("mathjaxEditing", function () {
    StackExchange.MarkdownEditor.creationCallbacks.add(function (editor, postfix) {
    StackExchange.mathjaxEditing.prepareWmdForMathJax(editor, postfix, [["$", "$"], ["\\(","\\)"]]);
    });
    });
    }, "mathjax-editing");

    StackExchange.ready(function() {
    var channelOptions = {
    tags: "".split(" "),
    id: "69"
    };
    initTagRenderer("".split(" "), "".split(" "), channelOptions);

    StackExchange.using("externalEditor", function() {
    // Have to fire editor after snippets, if snippets enabled
    if (StackExchange.settings.snippets.snippetsEnabled) {
    StackExchange.using("snippets", function() {
    createEditor();
    });
    }
    else {
    createEditor();
    }
    });

    function createEditor() {
    StackExchange.prepareEditor({
    heartbeatType: 'answer',
    autoActivateHeartbeat: false,
    convertImagesToLinks: true,
    noModals: true,
    showLowRepImageUploadWarning: true,
    reputationToPostImages: 10,
    bindNavPrevention: true,
    postfix: "",
    imageUploader: {
    brandingHtml: "Powered by u003ca class="icon-imgur-white" href="https://imgur.com/"u003eu003c/au003e",
    contentPolicyHtml: "User contributions licensed under u003ca href="https://creativecommons.org/licenses/by-sa/3.0/"u003ecc by-sa 3.0 with attribution requiredu003c/au003e u003ca href="https://stackoverflow.com/legal/content-policy"u003e(content policy)u003c/au003e",
    allowUrls: true
    },
    noCode: true, onDemand: true,
    discardSelector: ".discard-answer"
    ,immediatelyShowMarkdownHelp:true
    });


    }
    });














    draft saved

    draft discarded


















    StackExchange.ready(
    function () {
    StackExchange.openid.initPostLogin('.new-post-login', 'https%3a%2f%2fmath.stackexchange.com%2fquestions%2f3154792%2fwhy-int-2-infty1-over-sqrt-x-sqrt-x-12dx-styletext-decorationline-t%23new-answer', 'question_page');
    }
    );

    Post as a guest















    Required, but never shown

























    1 Answer
    1






    active

    oldest

    votes








    1 Answer
    1






    active

    oldest

    votes









    active

    oldest

    votes






    active

    oldest

    votes









    2












    $begingroup$

    The discrepancy comes mostly from the first two terms: the integral from $2$ to $3$ is about $2.1$ while the sum term for $n=2$ is about $4.2$ Same from $3$ to $4$ integral is about $.73$, the $n=3$ sum term is about $1.1$ so if you start from $4$ on and noting that because the function is decreasing, the integral is always smaller than the sum starting from same bound, you get the integral to be about $2$ and the sum to be about $2.17$ so the discrepancy gets much smaller.






    share|cite|improve this answer









    $endgroup$


















      2












      $begingroup$

      The discrepancy comes mostly from the first two terms: the integral from $2$ to $3$ is about $2.1$ while the sum term for $n=2$ is about $4.2$ Same from $3$ to $4$ integral is about $.73$, the $n=3$ sum term is about $1.1$ so if you start from $4$ on and noting that because the function is decreasing, the integral is always smaller than the sum starting from same bound, you get the integral to be about $2$ and the sum to be about $2.17$ so the discrepancy gets much smaller.






      share|cite|improve this answer









      $endgroup$
















        2












        2








        2





        $begingroup$

        The discrepancy comes mostly from the first two terms: the integral from $2$ to $3$ is about $2.1$ while the sum term for $n=2$ is about $4.2$ Same from $3$ to $4$ integral is about $.73$, the $n=3$ sum term is about $1.1$ so if you start from $4$ on and noting that because the function is decreasing, the integral is always smaller than the sum starting from same bound, you get the integral to be about $2$ and the sum to be about $2.17$ so the discrepancy gets much smaller.






        share|cite|improve this answer









        $endgroup$



        The discrepancy comes mostly from the first two terms: the integral from $2$ to $3$ is about $2.1$ while the sum term for $n=2$ is about $4.2$ Same from $3$ to $4$ integral is about $.73$, the $n=3$ sum term is about $1.1$ so if you start from $4$ on and noting that because the function is decreasing, the integral is always smaller than the sum starting from same bound, you get the integral to be about $2$ and the sum to be about $2.17$ so the discrepancy gets much smaller.







        share|cite|improve this answer












        share|cite|improve this answer



        share|cite|improve this answer










        answered Mar 20 at 0:53









        ConradConrad

        1,32745




        1,32745






























            draft saved

            draft discarded




















































            Thanks for contributing an answer to Mathematics Stack Exchange!


            • Please be sure to answer the question. Provide details and share your research!

            But avoid



            • Asking for help, clarification, or responding to other answers.

            • Making statements based on opinion; back them up with references or personal experience.


            Use MathJax to format equations. MathJax reference.


            To learn more, see our tips on writing great answers.




            draft saved


            draft discarded














            StackExchange.ready(
            function () {
            StackExchange.openid.initPostLogin('.new-post-login', 'https%3a%2f%2fmath.stackexchange.com%2fquestions%2f3154792%2fwhy-int-2-infty1-over-sqrt-x-sqrt-x-12dx-styletext-decorationline-t%23new-answer', 'question_page');
            }
            );

            Post as a guest















            Required, but never shown





















































            Required, but never shown














            Required, but never shown












            Required, but never shown







            Required, but never shown

































            Required, but never shown














            Required, but never shown












            Required, but never shown







            Required, but never shown







            Popular posts from this blog

            Magento 2 - Add success message with knockout Planned maintenance scheduled April 23, 2019 at 23:30 UTC (7:30pm US/Eastern) Announcing the arrival of Valued Associate #679: Cesar Manara Unicorn Meta Zoo #1: Why another podcast?Success / Error message on ajax request$.widget is not a function when loading a homepage after add custom jQuery on custom themeHow can bind jQuery to current document in Magento 2 When template load by ajaxRedirect page using plugin in Magento 2Magento 2 - Update quantity and totals of cart page without page reload?Magento 2: Quote data not loaded on knockout checkoutMagento 2 : I need to change add to cart success message after adding product into cart through pluginMagento 2.2.5 How to add additional products to cart from new checkout step?Magento 2 Add error/success message with knockoutCan't validate Post Code on checkout page

            Fil:Tokke komm.svg

            Where did Arya get these scars? Unicorn Meta Zoo #1: Why another podcast? Announcing the arrival of Valued Associate #679: Cesar Manara Favourite questions and answers from the 1st quarter of 2019Why did Arya refuse to end it?Has the pronunciation of Arya Stark's name changed?Has Arya forgiven people?Why did Arya Stark lose her vision?Why can Arya still use the faces?Has the Narrow Sea become narrower?Does Arya Stark know how to make poisons outside of the House of Black and White?Why did Nymeria leave Arya?Why did Arya not kill the Lannister soldiers she encountered in the Riverlands?What is the current canonical age of Sansa, Bran and Arya Stark?